Đến nội dung

Nguyenhuyen_AG

Nguyenhuyen_AG

Đăng ký: 09-09-2010
Offline Đăng nhập: 10-01-2019 - 16:22
****-

#386940 $(\frac{a-b}{b-c})^{2}+(\frac...

Gửi bởi Nguyenhuyen_AG trong 15-01-2013 - 15:58

Cho mình hỏi cái ý tưởng để nghĩ ra hằng đẳng thức đó?? :wacko:

Ta có thể lí giải điều này như sau.

Đặt $x=a-b,\; y=b-c,\; z=c-z.$ Ta có $x+y+z=0$ bất đẳng thức cần chứng minh trở thành $$\frac{x^2}{y^2}+\frac{y^2}{z^2}+\frac{z^2}{x^2}\ge 5.$$ Chú ý rằng $$\frac{x}{y}+\frac{y}{z}+\frac{z}{x}+\frac{x}{z}+\frac{y}{x}+\frac{z}{x}=\frac{x+y}{z}+\frac{y+z}{x}+\frac{z+x}{y}=-1-1-1=-3.$$ Nên ta có $$\begin{aligned}\frac{x^2}{y^2}+\frac{y^2}{z^2}+\frac{z^2}{x^2}-5&=\frac{x^2}{y^2}+\frac{y^2}{z^2}+\frac{z^2}{x^2}-6+1\\&=\left [\frac{x^2}{y^2}+\frac{y^2}{z^2}+\frac{z^2}{x^2}+2\left ( \frac{x}{z}+\frac{y}{x}+\frac{z}{x} \right ) \right ]+2\left ( \frac{x}{y}+\frac{y}{z}+\frac{z}{x} \right )+1\\&=\left ( \frac{x}{y}+\frac{y}{z}+\frac{z}{x} \right )^2+2\left ( \frac{x}{y}+\frac{y}{z}+\frac{z}{x} \right )+1\\&=\left ( \frac{x}{y}+\frac{y}{z}+\frac{z}{x}+1 \right )^2.\end{aligned}$$ Từ đó thu được đẳng thức đẹp như trên.


#386054 $(\frac{a-b}{b-c})^{2}+(\frac...

Gửi bởi Nguyenhuyen_AG trong 12-01-2013 - 20:25

Cho a,b,c là các số thực thỏa mãn:
(a-b)(b-c)(c-a) khác 0 CMR ta luôn có:
$(\frac{a-b}{b-c})^{2}+(\frac{b-c}{c-a})^{2}+(\frac{c-a}{a-b})^{2}\geq 5$
______________________________________________
bài này ngoài cách trong sách của anh Cẩn còn cách nào khác không??

Ta có đẳng thức sau đây $$\left (\frac{a-b}{b-c} \right)^{2}+\left(\frac{b-c}{c-a}\right)^{2}+\left (\frac{c-a}{a-b} \right )^{2}\geq 5+\left ( 1+ \frac{a-b}{b-c}+\frac{b-c}{c-a}+\frac{c-a}{a-b}\right )^2.$$ Từ đó dễ dàng suy ra điều phải chứng minh.


#381074 $\frac{z-xy}{x^{2}+xy+y^{2}...

Gửi bởi Nguyenhuyen_AG trong 27-12-2012 - 23:52

Bài toán 2: Cho $a,b,c\in R^{+}$, thỏa $ab+bc+ca=1$. Chứng minh rằng

$$\frac{1}{a+b}+\frac{1}{b+c}+\frac{1}{c+a}-\frac{1}{a+b+c}\geq 2$$

Bất đẳng thức trên tương đương với $$\frac{1}{a+b}+\frac{1}{b+c}+\frac{1}{c+a}\geq 2+\frac{1}{a+b+c}.$$ Chú ý rằng $$\frac{a+b+c}{a+b}=1+\frac{c}{a+b},$$ nên bất đẳng thức trên có thể viết lại như sau $$\frac{a}{b+c}+\frac{b}{c+a}+\frac{c}{a+b}+2\ge 2(a+b+c).$$ Sử dụng bất đẳng thức Cauchy-Schwarz, ta có $$\frac{a}{b+c}+\frac{b}{c+a}+\frac{c}{a+b}\ge \frac{(a+b+c)^2}{2(ab+bc+ca)}=\frac{(a+b+c)^2}{2}.$$ Vậy ta cần chứng minh $$\frac{(a+b+c)^2}{2}+2\ge 2(a+b+c).$$ Hiển nhiên đúng theo bát đẳng thức AM-GM.


#379115 $$5 \sqrt{a+b}+2c^2 \ge \frac{21...

Gửi bởi Nguyenhuyen_AG trong 20-12-2012 - 19:12

Bài toán 1.
Cho các số thực $a, b, c$ thỏa mãn $a+b \ge 0$ và $ab+bc+ca=6.$ Chứng minh rằng: $$5 \sqrt{a+b}+2c^2 \ge \frac{21}{2}$$


Bài toán này đã từng được thảo luận ở đây.


#378255 CMR $$(9(a^2+b^2+c^2)(ab+bc+ca)\ge (a+b+c)^2$$

Gửi bởi Nguyenhuyen_AG trong 17-12-2012 - 14:23

Cho $a,b,c$ là cách cạnh của tam giác CMR
$$9(a^2+b^2+c^2)(ab+bc+ca)\ge (a+b+c)^2$$

Bất đẳng thức mạnh hơn sau đây vẫn đúng $$[7(a^2+b^2+c^2)+2(ab+bc+ca)](ab+bc+ca) \ge (a+b+c)^4.$$ Và nó cũng hiển nhiên vì tương đương với $$(a^2+b^2+c^2-ab-bc-ca)[2(ab+bc+ca)-a^2-b^2-c^2]\ge0.$$


#375586 $\frac{(a+b+c)^3}{abc}\geq 25(a^2+b^2+c^2)...

Gửi bởi Nguyenhuyen_AG trong 06-12-2012 - 18:33

Lời giải của toán này đã có ở đây.


#375583 Với mọi $a,b,c$ CMR: $$(a+b)^4+(b+c)^4+(c+a)^4\ge...

Gửi bởi Nguyenhuyen_AG trong 06-12-2012 - 18:27

Với mọi $a,b,c$ CMR:
$$(a+b)^4+(b+c)^4+(c+a)^4\ge \frac{4}{7}(a^4+b^4+c^4)$$

Bài toán mạnh hơn sau đây vẫn đúng $$(a+b)^4+(b+c)^4+(c+a)^4\ge \frac{4}{7}[a^4+b^4+c^4+(a+b+c)^4]$$


#375079 $\frac{x+y}{y+z}+\frac{y+z}...

Gửi bởi Nguyenhuyen_AG trong 04-12-2012 - 18:20

Mình xin giải bài toán 1:
Đặt a=x+y,b=y+z,c=z+x
BĐT<=>$\frac{a}{b}+\frac{b}{c}+\frac{c}{a}\geq \frac{b+c}{a+b}+\frac{a+2b+c}{b+c}$

Sau không dùng Cauchy-Schwarz cho nó gọn :D

Ta có bất đẳng thức trên tương đương với $$\frac{a}{b}+\frac{b}{c}+\frac{c}{a}\geq \frac{b+c}{a+b}+\frac{a+b}{b+c}+1,$$ $$\frac{a}{b}+\frac{b}{c}+\frac{c}{a}+1\geq \frac{(a+2b+c)^2}{(a+b)(b+c)},$$ Sử dụng bất đẳng thức Cauchy-Schwarz, ta có $$Vt=\frac{a^2}{ab}+\frac{b^2}{bc}+\frac{c^2}{ca}+\frac{b^2}{b^2}\geq \frac{(a+b+c+b)^2}{ab+bc+ca+b^2} = \frac{(a+2b+c)^2}{(a+b)(b+c)}.$$ Chứng minh hoàn tất.

Chú ý rằng, bất đẳng thức trên còn tương đương với $$\frac{a}{b}+\frac{b}{c}+\frac{c}{a} \ge 3+\frac{(c-a)^2}{ab+bc+ca+b^2}.$$ Vì $ab+bc+ca < ab+bc+ca+b^2,$ nên ta sẽ đưa bài toán về chứng minh bất đẳng thức mạnh hơn sau đây $$\frac{a}{b}+\frac{b}{c}+\frac{c}{a} \ge 3+\frac{(c-a)^2}{ab+bc+ca}.$$ Đây là một bài toàn quen thuộc của anh Cẩn có thể chứng minh bằng AM-GM, ngoài ra thì $k=1$ cũng là số lớn nhất để $$\frac{a}{b}+\frac{b}{c}+\frac{c}{a} \ge 3+\frac{k(c-a)^2}{ab+bc+ca}.$$ Bất đẳng thức sau luôn đúng. :D


#373071 $T\cdot a_1a_2\cdot\cdot\cdot a_n > a_1^n+a_2^n+...

Gửi bởi Nguyenhuyen_AG trong 27-11-2012 - 18:34

Với $a_1,\;a_2,\;...,\;a_n\; (n \ge 3)$ là các số thực dương. Hãy tìm giá trị nhỏ nhất của $T$ sao cho nếu ta có bất đẳng thức $$T\cdot a_1a_2\cdot\cdot\cdot a_n > a_1^n+a_2^n+\cdot\cdot\cdot+a_n^n,$$ thì $a_i,\;a_j,\; a_k$ là độ dài ba cạnh của tam giác trong đó $i,\;j,\;k$ là các số tự nhiên thỏa mãn điều kiện $0<i<j<k\le n.$


#370033 $$a^3+b^3+c^3+9abc\leq 2.\left[ab(a+b)+bc(b+c)+ca(c+a)...

Gửi bởi Nguyenhuyen_AG trong 16-11-2012 - 23:30

Bài toán 1.
Ch0 $a,b,c$ là 3 cạnh của 1 tam giác.Chứng minh rằng:
$$a^3+b^3+c^3+9abc\leq 2.\left[ab(a+b)+bc(b+c)+ca(c+a)\right]$$

Bài này có thể dùng S-S để giải, khá đơn giản. Thật vậy, do tính đối xứng của bài toán nên ta có thể giả sử $c=\max\{a,b,c\}.$ Khi đó bằng cách sử dụng các phân tích $$\begin{aligned}a^3+b^3+c^3-3abc&=(a+b+c)[(a-b)^2+(a-c)(b-c)]\\ab(a+b)+bc(b+c)+ca(c+a)-6abc&=2c(a-b)^2+(a+b)(a-c)(b-c).\end{aligned}$$ Ta có thể viết bất đẳng thức trên lại như sau $$2[ab(a+b)+bc(b+c)+ca(c+a)-6abc] \ge a^3+b^3+c^3-3abc,$$ $$4c(a-b)^2+2(a+b)(a-c)(b-c) \ge (a+b+c)[(a-b)^2+(a-c)(b-c)],$$ $$(3c-a-b)(a-b)^2+(a+b-c)(a-c)(b-c) \ge 0.$$ Điều này là luôn đúng theo giả thiết của bài toán.
Nhận xét : Ngoài ra ta có thể chứng minh bài toán này bằng phép thế Ravi. Sử dụng bài toán này, ta có thể chứng minh được bài toán đây sau từng xuất hiện trên THTT (lời giải trên tạp chí sử dụng đến công cụ hình học) $$\frac{a}{\sqrt{a^2+3bc}}+\frac{b}{\sqrt{b^2+3ca}}+\frac{c}{\sqrt{c^2+3ca}}\ge\frac{3}{2}.$$


#368999 $ \sum\limits_{k = 1}^n {\frac{{...

Gửi bởi Nguyenhuyen_AG trong 12-11-2012 - 19:30

Các bạn thử tìm lời giải bài toán này bằng Cauchy-SChwarz xem. :D


#368535 $$\frac{x^2-y^2}{y}+\frac{y^2-z^...

Gửi bởi Nguyenhuyen_AG trong 10-11-2012 - 21:28

Bài toán 3:
Cho $a, b, c>0$ thỏa mãn $a <b \le c \le 1$ và $30a+4b+c \le 34$. Tìm GTLN của: $$P=30a^2+4b^2+c^2$$

(Đề OLP 30-4 THPT chuyên Trà Vinh 2012)

Từ giả thiết của bài toán, ta có $$30a\le 34-4b-c \Rightarrow 30a^2\le a(34-4b-c)=34a-4ab-ca.$$ Từ đó suy ra $$\begin{aligned} P=30a^2+4b^2+c^2&\le 34a-4ab-ca+4b^2+c^2\\&=34a+4b(b-a)+c(c-a)\\&=34a+4(b-a)+(c-a)\\&\le 29a+4b+c.\end{aligned}$$ Như vậy, ta được $$P\le 29a+4b+c.$$ Mặt khác, theo bất đẳng thức Cauchy-Schwarz, thì $$P^2\le (29a+4b+c)^2\le \left ( \frac{841}{30}+4+1 \right )(30a^2+4b^2+c^2)=\frac{991}{30}\cdot P$$ Như vâỵ $P\le \frac{991}{30},$ ngoài ra với $a=\frac{29}{30},b=c=1,$ thì $30a+4b+c=34$ và $P= \frac{991}{30}.$ Điều này cho phép ta kết luận $P_{\max}= \frac{991}{30}.$ :D


#368510 $$\frac{x^2-y^2}{y}+\frac{y^2-z^...

Gửi bởi Nguyenhuyen_AG trong 10-11-2012 - 20:34

Bài toán 1:
Cho $x, y, z$ là các số thực dương. Chứng minh rằng: $$\frac{x^2-y^2}{y}+\frac{y^2-z^2}{z}+\frac{z^2-x^2}{x} \ge \frac{(x-y)^2}{y+z}+\frac{(y-z)^2}{z+x}+\frac{(z-x)^2}{x+y}$$

(Đề OLP 30-4 THPT chuyên Bến Tre 2012)

Bài này không cần phải dùng đến công cụ mạnh. Chú ý rằng $$\frac{(x-y)^2}{y}+\frac{(y-z)^2}{z}+\frac{(z-x)^2}{x}\ge \frac{(x-y)^2}{y+z}+\frac{(y-z)^2}{z+x}+\frac{(z-x)^2}{x+y},$$ nên ta sẽ đi chứng minh bất đẳng thức mạng hơn sau đây $$\frac{x^2-y^2}{y}+\frac{y^2-z^2}{z}+\frac{z^2-x^2}{x}\ge\frac{(x-y)^2}{y}+\frac{(y-z)^2}{z}+\frac{(z-x)^2}{x}.$$ Thế nhưng đây chỉ là một đẳng thức vì ta luôn có $$\frac{x^2-y^2}{y}+\frac{y^2-z^2}{z}+\frac{z^2-x^2}{x}=\frac{(x-y)^2}{y}+\frac{(y-z)^2}{z}+\frac{(z-x)^2}{x}.$$ Chứng minh hoàn tất. :D


#366625 $x = 3\sqrt {x + 1} + 3\sqrt {y + 2} - y...

Gửi bởi Nguyenhuyen_AG trong 02-11-2012 - 20:15

Tìm Min của P = x + y với x và y là 2 số thực thỏa mãn:
$x = 3\sqrt {x + 1} + 3\sqrt {y + 2} - y$

Đây là đề thi Học sinh giỏi quốc gia năm 2005, ngoài lời giải dưới đây thì bạn có thể sử dụng phương pháp miền giá trị.

Let $X=\sqrt{x+1},Y=\sqrt{y+2}\Longleftrightarrow x=X^2-1,y=Y^2-2$.We can rewrite the given conditions to $X^2+Y^2-3=3(X+Y)\ (X\geqq 0,Y\geqq 0)$ and$P=3(X+Y)$

$\Longleftrightarrow \left(X-\frac{3}{2}\right)^2+\left(Y-\frac{3}{2}\right)^2=\frac{15}{2}\ (X\geqq0,Y\geqq0)$ and $X+Y=\frac{P}{3}$

Determining the condition for which a circle with centered $\left(\frac{3}{2},\frac{3}{2}\right)$,radius $\frac{\sqrt{15}}{\sqrt{2}}$ and a line $X+Y=\frac{P}{3}$ have at least of the interception point,

Max$P$ is $9+3\sqrt{15}$, when $(X,Y)=\left(\frac{3+\sqrt{15}}{2},\frac{3+\sqrt{15}}{2}\right)$

$\Longleftrightarrow (x,y)=\left(\frac{10+3\sqrt{15}}{2},\frac{8+3\sqrt{5}}{2}\right)$, and

min$P$ is $\frac{9+3\sqrt{21}}{2}$,when $(X,Y)=\left(\frac{3+\sqrt{21}}{2},0\right),\left(0,\frac{3+\sqrt{21}}{2}\right)$

$\Longleftrightarrow (x,y)=\left(\frac{13+3\sqrt{21}}{2},-2\right),\left(-2,\frac{13+3\sqrt{21}}{2}\right)$.




#366132 $$\frac{1}{a}+\frac{1}...

Gửi bởi Nguyenhuyen_AG trong 31-10-2012 - 11:57

Bất đẳng thức trên ương đương với $$ab+bc+ca+\frac{6}{a+b+c}\ge 5.$$ Theo nguyên lí Dirichlet ta có thể giả sử $(b-1)(c-1) \ge 0,$ tương đương với $b+c \le 1+bc.$ VKhi đó, ta thu được $$a+b+c\le a+1+bc=a+1+\frac{1}{a}=\frac{a^2+a+1}{a}.$$ Cà theo bất đẳng thức AM-GM thì $$ab+bc+ca=a(b+c)+bc\ge 2a\sqrt{bc}+bc=2\sqrt{a}+\frac{1}{a}.$$ Từ hai đánh giá trên, ta có $$ab+bc+ca+\frac{6}{a+b+c}\ge 2\sqrt{a}+\frac{1}{a}+\frac{6a}{a^2+a+1}.$$ Đặt $t=\sqrt{a}$ ta sẽ chứng minh $$2t+\frac{1}{t}+\frac{6t^2}{t^4+t^2+1} \ge 5.$$ Bằng cachs quy đồng và thi gọn, ta thấy bất đẳng thức này tương đương với $$(t-1)^2[t(t-1)^2(2t^2+3t+2)+1] \ge 0.$$ Luôn đúng, vậy ta có điều phải chứng minh